11
$\begingroup$

Let $G$ be a countable group. A Følner sequence is a sequence of finite subsets $(F_n)_n$ such that

$$\lim_{n\to\infty} \frac{|KF_n \mathbin\triangle F_n|}{|F_n|} = 0$$

for each fixed finite subset $K \subset G$. Such a sequence exists if and only if the group is amenable. The sequence is said to satisfy Tempelman's condition if there is a constant $C \geq 1$ such that $|F_n^{-1}F_n| \leq C|F_n|$ for every $n$. Let us say $(F_n)_n$ is a Tempelman sequence if it is a Følner sequence satisfying Tempelman's condition. Not every countable amenable group admits a Tempelman sequence; see Hochman - Averaging sequences and abelian rank in amenable groups. Every countable nilpotent group has a Tempelman sequence.

My question is:

If $(F_n)_n$ is a Tempelman sequence, is the sequence $(F_n^{-1}F_n)_n$ necessarily a Følner sequence? Or: can a Tempelman sequence $(F_n)_n$ always be found such that $(F_n^{-1}F_n)_n$ is Følner?

This is obviously true in $\mathbb{Z}^d$, where if $F_n = [0,n)^d \subset \mathbb{Z}^d$ then $F_n^{-1}F_n = (-n,n)^d$. It seems like it should be true in general; if $K$ is fixed and $n$ is large then intuitively $|KF^{-1}_nF_n \mathbin\triangle F_n^{-1}F_n| \lesssim C|KF_n \mathbin\triangle F_n|$ should follow from $|F_n^{-1}F_n| \leq C|F_n|$. Yet, I cannot find a proof.

I can see how it is done under a modified condition: suppose for every $n$ there is a subset $C_n \subset G$ such that $|C_n| \leq C$ and $F_n^{-1}F_n \subset F^{-1}_nC_n$. If it makes things easier, we can assume that $F_n^{-1} = F_n$ (in which case $(F_n)_n$ is both left Følner and right Følner).

Proof: assume that $e\in K$ and $F = F_n = F_n^{-1}$ is large. Any element $g \in KFF \setminus FF$ decomposed as $g = kf_1f_2$ in any way necessarily has $kf_1 \notin F$. Then, $FF \subset FC$ implies each $g \in KFF\setminus FF$ may be decomposed as $g = kfc$. There is therefore a map $g \mapsto (kf,c)$ which embeds $KFF \setminus FF$ in $(KF \setminus F) \times C$; the result follows.

$\endgroup$
2
  • $\begingroup$ Unless I'm mistaken, you specify in the title but not the body that $(F_n)_n$ is a Følner sequence. $\endgroup$ Aug 20, 2022 at 12:52
  • $\begingroup$ Thank you; I have edited the question to add that assumption $\endgroup$
    – Rob
    Aug 20, 2022 at 15:58

1 Answer 1

12
$\begingroup$

The answer to the first question is negative, even for the integers $G = \mathbb{Z}$. The point is that Følner sequences $F_n$ are stable under modification by small sets, but the difference set $F_n^{-1} F_n$ can be significantly affected by such a modification. For a concrete example, take

$$ F_n = \{0,\dots,n^2\} \cup \{ 2n^2 + 2nj: 0 \leq j < n \} \cup \{ -2n^2 - 2i: 0 \leq i < n\}.$$ This is a modification of $\{0,\dots,n^2\}$ by $O(n)$ elements and is thus a Følner sequence; it is also a dense subset of $\{-4n^2,\dots,4n^2\}$ and thus obeys the Tempelman condition. On the other hand, $F_n^{-1} F_n$ has cardinality $\asymp n^2$ and contains the even elements of $\{4n^2,\dots,6n^2-2\}$ but not the odd elements, so will not be Følner. (This counterexample is not symmetric, but it is not difficult to modify the construction to give a symmetric counterexample; we leave this as an exercise to the interested reader.)

On the other hand, given a Følner sequence $F_n$ obeying the Tempelman condition, one can use a standard pigeonholing argument to find a sequence of subsets $F'_n$ of $F_n$ with $|F_n \backslash F'_n| = o(|F_n|)$ (so that $F'_n$ is also a Følner sequence and will also obey the Tempelman condition) such that ${F'}_n^{-1} {F'}_n$ is Følner; in the above example, this would amount to "trimming" the unwanted portions $\{ 2n^2 + 2nj: 0 \leq j < n \} \cup \{ -2n^2 - 2i: 0 \leq i < n\}$ of $F_n$ and only keeping the "core" portion $\{0,\dots,n^2\}$. A sketch of the construction is as follows. Let $K \subset G$ be a finite symmetric neighborhood of the identity and let $k \geq 2$ be a natural number. For $n$ large enough one can use the Følner condition to find $F''_n \subset F_n$ with $|F_n \backslash F''_n| \leq \frac{1}{k} |F_n|$ and $F''_n \cdot K^k \subset F_n$. The sets $K^i \cdot (F''_n)^{-1} \cdot F''_n \cdot K^{i}$ for $i=0,\dots,k$ are then increasing with cardinality at most $|F_n^{-1} \cdot F_n| \leq C |F_n|$, so by the pigeonhole principle one can find $0 \leq i < k$ such that $$ |K^{i+1} \cdot (F''_n)^{-1} \cdot F''_n \cdot K^{i+1}| \leq |K^{i} \cdot (F''_n)^{-1} \cdot F''_n \cdot K^{i}| + \frac{C}{k} |F_n|.$$ If we then set $F'_n := F''_n \cdot K^i$ then $F'_n$ is a subset of $F_n$ with $|F_n \backslash F'_n| \leq \frac{1}{k} |F_n|$ and $$ |(K \cdot (F'_n)^{-1} \cdot F'_n) \backslash ((F'_n)^{-1} \cdot F'_n)| \leq \frac{C}{k} |F_n|.$$ From this it is an easy matter to verify that $(F'_n)^{-1} \cdot F'_n$ will obey the Følner condition (letting $K, k$ grow sufficiently slowly with $n$).

$\endgroup$
1
  • $\begingroup$ Thank you very much! This really illuminates the problem for me. $\endgroup$
    – Rob
    Aug 20, 2022 at 21:57

Your Answer

By clicking “Post Your Answer”, you agree to our terms of service and acknowledge you have read our privacy policy.

Not the answer you're looking for? Browse other questions tagged or ask your own question.